Feat: Annales pour les TESL

This commit is contained in:
Bertrand Benjamin 2020-05-15 08:44:40 +02:00
parent cd8f2b3399
commit ca23051441
3 changed files with 390 additions and 0 deletions

Binary file not shown.

View File

@ -0,0 +1,26 @@
\documentclass[a4paper,10pt]{article}
\usepackage{myXsim}
\title{Logarithme - annales}
\tribe{Terminale TESL}
\date{Mai 2020}
\pagestyle{empty}
\geometry{left=10mm,right=10mm, top=10mm}
\renewcommand{\baselinestretch}{0.8}
\DeclareExerciseCollection{banque}
\xsimsetup{
step=4,
%solution/print=true,
}
\begin{document}
\maketitle
\input{banque.tex}
\printcollection{banque}
\end{document}

View File

@ -167,4 +167,368 @@
\end{enumerate}
\end{enumerate}
\end{exercise}
\begin{exercise}[subtitle={Coût de fabrication}, step={4}, topics={Logarithme}]
% Polynésie Juin 2019 Ex 4
\begin{center}\textbf{Les deux parties de cet exercice sont indépendantes}\end{center}
\textbf{Partie A}
\medskip
Une entreprise produit chaque année entre $100$ et $900$ pneus pour tracteurs.
On considère la fonction $f$ définie sur l'intervalle [1~;~9] par
\[f(x ) = 0,5 x^2 - 7x + 14 + 6\ln (x).\]
On admet que la fonction $f$ modélise le coût moyen annuel de fabrication d'un pneu, exprimé en centaines d'euros, pour $x$ centaines de pneus produits.
\medskip
\begin{enumerate}
\item La fonction $f$ est dérivable sur l'intervalle [1~;~9] et on note $f'$ sa fonction dérivée.
Démontrer que pour tout réel $x$ de l'intervalle [1~;~9] on a : $f'(x)= \dfrac{x^2 -7 x + 6}{x}$.
\item
\begin{enumerate}
\item Justifier les variations suivantes de la fonction $f$ sur l'intervalle [1~;~9] :
\begin{center}
\begin{tikzpicture}[baseline=(a.north)]
\tkzTabInit[lgt=3,espcl=3]{$x$/1,Variations de\\ $f(x)$/2}{1, 6, 9}
\tkzTabVar{+/ , -/ , +/ }
\end{tikzpicture}
\end{center}
\item Justifier que, sur l'intervalle [1~;~9], l'équation $f(x) = 5$ admet une unique solution $\alpha$.
\item Donner un encadrement au centième près de $\alpha$.
\item On considère l'algorithme ci-dessous:
\begin{center}
\begin{tabularx}{0.5\linewidth}{|X|}\hline
$X \gets 1$\\
$Y \gets 7,5$\\
Tant que $Y > 5$\\
\hspace{12mm}$X \gets X + 0,01$\\
\hspace{12mm}$Y \gets 0,5X^2 - 7X + 14 + 6*\ln (X)$\\
Fin Tantque\\ \hline
\end{tabularx}
\end{center}
À la fin de l'exécution de l'algorithme, quelle valeur numérique contient la variable $X$?
\end{enumerate}
\item Pour quelle quantité de pneus, le coût moyen annuel de fabrication d'un pneu est-il minimal ?
À combien s'élève-t-il ?
\end{enumerate}
\bigskip
\textbf{Partie B}
\medskip
Cette même entreprise envisage la fabrication de semoirs (gros matériel agricole).
On admet que la fonction $g$ définie sur l'intervalle [0~;~100] par
\[g (x) = 2x -1 + \text{e}^{0,05x}\]
modélise le coût de fabrication, exprimé en centaines d'euros, de $x$ semoirs.
\medskip
\begin{enumerate}
\item Donner une primitive $G$ de la fonction $g$ sur l'intervalle [0~;~100].
\item Calculer la valeur moyenne de la fonction $g$ sur l'intervalle [0~;~100].
\item Interpréter ce résultat dans le contexte de l'exercice.
\end{enumerate}
\end{exercise}
\begin{solution}
\textbf{Partie A}
\medskip
Une entreprise produit chaque année entre $100$ et $900$ pneus pour tracteurs.
On considère la fonction $f$ définie sur l'intervalle $\left [1~;~9\strut\right ]$ par
$f(x ) = 0,5 x^2 - 7x + 14 + 6\ln (x)$.
On admet que la fonction $f$ modélise le coût moyen annuel de fabrication d'un pneu, exprimé en centaines d'euros, pour $x$ centaines de pneus produits.
\medskip
\begin{enumerate}
\item% La fonction $f$ est dérivable sur l'intervalle $\left [1~;~9\strut\right ]$ et on note $f'$ sa fonction dérivée.
Pour tout réel $x$ de l'intervalle $\left [1~;~9\strut\right ]$ on a
$f(x ) = 0,5 x^2 - 7x + 14 + 6\ln (x)$ donc\\
$f'(x)= 0,5\times 2x - 7 + 6\times \dfrac{1}{x}
= x-7-\dfrac{6}{x}
= \dfrac{x^2-7x+6}{x}$.
%Démontrer que pour tout réel $x$ de l'intervalle $\left [1~;~9\strut\right ]$ on a : $f'(x)= \dfrac{x^2 -7 x + 6}{x}$.
\item
\begin{enumerate}
\item On va justifier les variations suivantes de la fonction $f$ sur l'intervalle [1~;~9] :
\begin{center}
{\renewcommand{\arraystretch}{1.1}
\psset{nodesep=3pt,arrowsize=2pt 3} % paramètres
\def\esp{\hspace*{1.5cm}}% pour modifier la largeur du tableau
\def\hauteur{0pt}% mettre au moins 20pt pour augmenter la hauteur
$\begin{array}{|c| *4{c} c|}
\hline
x & 1 & \esp & 6 & \esp & 9 \\
\hline
& \Rnode{max1}{\phantom{0}} & & & & \Rnode{max2}{\phantom{0}} \\
\text{Variations de } f & & & & & \rule{0pt}{\hauteur} \\
& & & \Rnode{min}{\phantom{0}} & & \rule{0pt}{\hauteur}
\ncline{->}{max1}{min} \ncline{->}{min}{max2}
%\rput*(-3.7,0.65){\Rnode{zero}{\blue 0}}
%\rput(-3.7,1.7){\Rnode{alpha}{\blue \alpha}}
%\ncline[linestyle=dotted, linecolor=blue]{alpha}{zero}
%\rput*(-1.3,0.65){\Rnode{zero2}{\red 0}}
%\rput(-1.3,1.7){\Rnode{beta}{\red \beta}}
%\ncline[linestyle=dotted, linecolor=red]{beta}{zero2}
\\
\hline
\end{array}$
}
\end{center}
Sur $\left [1~;~9\strut\right ]$, $f'(x)$ est du signe de $x^2-7x+6$.
$\Delta = 7^2 - 4\times 1\times 6 = 49-24=25=5^2$ donc le trinôme $x^2-7x+6$ admet deux racines:\\[3pt]
$x'=\dfrac{-b-\sqrt{\Delta}}{2a} = \dfrac{7-5}{2}=1$ et $x''=\dfrac{-b+\sqrt{\Delta}}{2a}=\dfrac{7+5}{2}=6$.
\smallskip
On en déduit le signe du trinôme (positif à l'extérieur des racines) donc de $f'(x)$:
\begin{center}
\renewcommand{\arraystretch}{1.5}
\def\esp{\hspace*{1.2cm}}
$\begin{array}{|c | *{5}{c} |}
\hline
x & 1 & \esp & 6 & \esp & 9 \\
\hline
f'(x) & & \pmb{-} & \vline\hspace{-2.7pt}{0} & \pmb{+} & \\
\hline
\end{array}$
\end{center}
Cela justifie les variations de $f$.
\item% Justifier que, sur l'intervalle [1~;~9], l'équation $f(x) = 5$ admet une unique solution $\alpha$.
On complète le tableau de variations de $f$:
$f(1)=7,5$, $f(6)\approx 0,75$ et $f(9)\approx 4,7$
\begin{center}
{\renewcommand{\arraystretch}{1.3}
\psset{nodesep=3pt,arrowsize=2pt 3} % paramètres
\def\esp{\hspace*{1.5cm}}% pour modifier la largeur du tableau
\def\hauteur{0pt}% mettre au moins 20pt pour augmenter la hauteur
$\begin{array}{|c| *4{c} c|}
\hline
x & 1 & \esp & 6 & \esp & 9 \\
\hline
& \Rnode{max1}{7,5} & & & & \Rnode{max2}{\approx 4,7} \\
\text{Variations de } f & & & & & \rule{0pt}{\hauteur} \\
& & & \Rnode{min}{\approx 0,75} & & \rule{0pt}{\hauteur}
\ncline{->}{max1}{min} \ncline{->}{min}{max2}
\rput*(-4.5,0.65){\Rnode{zero}{\blue 5}}
\rput(-4.5,1.7){\Rnode{alpha}{\blue \alpha}}
\ncline[linestyle=dotted, linecolor=blue]{alpha}{zero}
\\
\hline
\end{array}$
}
\end{center}
On en déduit que sur $\left [1~;~9\strut\right ]$, l'équation $f(x)=5$ admet une solution unique $\alpha$.
\item %Donner un encadrement au centième près de $\alpha$.
$\left.
\begin{array}{l}
f(2)\approx 6,2 > 5\\
f(3)\approx 4,1 < 5
\end{array}
\right\rbrace
\Rightarrow
\alpha \in \left [ 2~;\,3\strut\right ]$
\hfill
$\left.
\begin{array}{l}
f(2,5)\approx 5,1 > 5\\
f(2,6)\approx 4,9 < 5
\end{array}
\right\rbrace
\Rightarrow
\alpha \in \left [ 2,5~;\, 2,6 \strut\right ]$
$\left.
\begin{array}{l}
f(2,55)\approx 5,018 > 5\\
f(2,56)\approx 4,997 < 5
\end{array}
\right\rbrace
\Rightarrow
\alpha \in \left [ 2,55~;\, 2,56 \strut\right ]$
\item On considère l'algorithme ci-dessous:
\begin{center}
\begin{tabularx}{0.5\linewidth}{|X|}\hline
$X \gets 1$\\
$Y \gets 7,5$\\
Tant que $Y > 5$\\
\hspace{12mm}$X \gets X + 0,01$\\
\hspace{12mm}$Y \gets 0,5X^2 - 7X + 14 + 6*\ln (X)$\\
Fin Tantque\\ \hline
\end{tabularx}
\end{center}
À la fin de l'exécution de l'algorithme, la variable $X$ contient la valeur $2,56$, première valeur au centième pour laquelle $Y>5$.
\end{enumerate}
\item Le coût moyen annuel de fabrication d'un pneu est minimal quand la fonction $f$ atteint son minimum c'est-à-dire pour $x=6$; c'est donc pour la fabrication de 600 pneus que le coût moyen annuel de fabrication d'un pneu est minimal. Ce coût est, en euro, de $f(6)\times 100 \approx 75$.
%À combien s'élève-t-il ?
\end{enumerate}
\bigskip
\textbf{Partie B}
\medskip
Cette même entreprise envisage la fabrication de semoirs (gros matériel agricole).
On admet que la fonction $g$ définie sur l'intervalle $\left [0~;~100\strut\right ]$ par
$g (x) = 2x -1 + \e^{0,05x}$
modélise le coût de fabrication, exprimé en centaines d'euros, de $x$ semoirs.
\medskip
\begin{enumerate}
\item %Donner une primitive $G$ de la fonction $g$ sur l'intervalle $\left [0~;~100\strut\right ]$.
Sur l'intervalle $\left [0~;~100\strut\right ]$, la fonction $g$ a pour primitive la fonction $G$ définie par\\
$G(x)=x^2 - x + \dfrac{\e^{0,05x}}{0,05} = x^2-x +20\e^{0,05x}$.
\item La valeur moyenne $m$ de la fonction $g$ sur l'intervalle $\left [0~;~100\strut\right ]$ est:
$m=\dfrac{1}{100-0} \ds\int_{0}^{100} g(x) \d x = \dfrac{1}{100} \left [G(100) - G(0) \strut\right ]
=\dfrac{1}{100}\left [ \left ( \np{9900} + 20\e^{5}\right ) - \left ( 20\right ) \right ]
= \np{9880} +20\e^{5}\\[3pt]
\phantom{m}
\approx \np{128,46}$
\item %Interpréter ce résultat dans le contexte de l'exercice.
Le coût moyen d'un semoir est donc, en euro, $128,46 \times 100 = \np{12846}$.
\end{enumerate}
\end{solution}
\begin{exercise}[subtitle={Étude de fonction}, step={4}, topics={Logarithme}]
% Métropole Septembre 2019 Ex 3
La courbe $\mathcal{C}_f$ ci-dessous est la courbe représentative d'une fonction $f$ définie et deux
fois dérivable sur l'intervalle [1,1~;~8].
\begin{center}
\begin{tikzpicture}[yscale=0.5, xscale=1.5, domain=1:8]
\tkzInit[xmin=0,xmax=9,xstep=1,
ymin=0,ymax=12,ystep=1]
\tkzGrid
\tkzAxeXY[up space=0.5,right space=.5]
\clip (1.1, 0) rectangle (9,12);
\tkzFct[line width=1pt]{(2.*x-1.)/log(x)}
\end{tikzpicture}
\end{center}
\textbf{Les parties A et B peuvent être traitées de manière indépendante.}
\bigskip
\textbf{Partie A : étude graphique}
\medskip
\begin{enumerate}
\item Donner une valeur approchée du minimum de la fonction $f$ sur l'intervalle
[1,1~;~8]
\item Quel est le signe de $f'(5)$ ? Justifier.
\item Encadrer l'intégrale $\displaystyle\int_2^4 f(x)\:\text{2}4 f(x)\:\text{d}x$ par deux entiers consécutifs.
\item La fonction $f$ est-elle convexe sur [1,1~;~3] ? Justifier.
\end{enumerate}
\bigskip
\textbf{Partie B : étude analytique}
\medskip
On admet que $f$ est la fonction définie sur l'intervalle [1,1~;~8] par
\[f(x) = \dfrac{2x - 1}{\ln (x)}.\]
\smallskip
\begin{enumerate}
\item Montrer que, pour tout réel $x$ de l'intervalle [1,1~;~8], on a :
\[f'(x) = \dfrac{2\ln (x) - 2 + \frac{1}{x}}{(\ln (x))^2}\]
\item Soit $h$ la fonction définie sur [1,1~;~8] par : $h(x) = 2\ln (x) - 2 + \frac{1}{x}$.
\begin{enumerate}
\item Soit $h'$ la fonction dérivée de $h$ sur l'intervalle [1,1; 8].
Montrer que, pour tout réel $x$ de l'intervalle [1,1~;~8],
\[h'(x) = \dfrac{2x - 1}{x^2}.\]
\item En déduire les variations de la fonction $h$ sur l'intervalle [1,1~;~8].
\item Montrer que l'équation $h(x) = 0$ admet une unique solution $\alpha$ sur
l'intervalle [1,1~;~8]. Donner un encadrement de $\alpha$ par deux entiers
consécutifs.
\end{enumerate}
\item Déduire des résultats précédents le signe de $h(x)$ sur l'intervalle [1,1~;~8].
\item À l'aide des questions précédentes, donner les variations de [ sur [1,1~;~8].
\end{enumerate}
\end{exercise}
\begin{exercise}[subtitle={Loi de Benfort}, step={4}, topics={Logarithme}]
% Métropole Juin 2017 Ex 4
Dans cet exercice, on considère le premier chiffre des entiers naturels non nuls, en écriture décimale. Par exemple, le premier chiffre de \np{2017} est 2 et le premier chiffre de 95 est 9.
Dans certaines circonstances, le premier chiffre d'un nombre aléatoire non nul peut être modélisé par une variable aléatoire $X$ telle que pour tout entier $c$ compris entre 1 et 9,
\[P(X = c) = \dfrac{\ln (c + 1) - \ln (c)}{\ln(10)}.\]
Cette loi est appelée loi de Benford.
\medskip
\begin{enumerate}
\item Que vaut $P(X = 1)$ ?
\item On souhaite examiner si la loi de Benford est un modèle valide dans deux cas particuliers.
\begin{enumerate}
\item \textbf{Premier cas}
Un fichier statistique de l'INSEE indique la population des communes en France au 1\ier{} janvier 2016 (champ: France métropolitaine et départements d'outre-mer de la Guadeloupe, de la Guyane, de la Martinique et de la Réunion).
À partir de ce fichier, on constate qu'il y a \np{36677} communes habitées. Parmi elles, il y a \np{11094} communes dont la population est un nombre qui commence par le chiffre 1.
Cette observation vous semble-t-elle compatible avec l'affirmation : \og{}le premier chiffre de la population des communes en France au 1 er janvier 2016 suit la loi de Benford \fg{} ?
\item \textbf{Deuxième cas}
Pour chaque candidat au baccalauréat de la session 2017, on considère sa taille en centimètres.
On désigne par $X$ la variable aléatoire égale au premier chiffre de la taille en centimètres d'un candidat pris au hasard.
La loi de Benford vous semble-t-elle une loi adaptée pour $X$ ?
\end{enumerate}
\end{enumerate}
\end{exercise}
\collectexercisesstop{banque}